Вы находитесь на странице: 1из 50

HANDOUT ON ADOLESCENT MEDICINE

PUBERTY - NORMAL GROWTH AND DEVELOPMENT


To assess growth and development during puberty it is critical to evaluate sexual
maturity ratings as these correlate to normal growth and also physiologic changes.
Sexual maturity ratings include breast development in females, genital
development in males and pubic hair development in both males and females.
KEY HORMONES
Many of the body's hormones influence growth, such as growth hormone, thyroxine,
insulin, and corticosteroids (all of which influence growth rate), leptin (which alters
body composition), and parathyroid hormone, 1,25-dihydroxy-vitamin D, and
calcitonin (all of which affect skeletal mineralization). However, the key hormone in
growth is GH which is mediated by growth hormone-releasing hormone (GHRH) and
somatostatin (SS). Growth hormone secretion is increased by GHRH and decreased
by somatostatin. Both growth hormone levels and IGF-I (somatomedin-C) levels rise
during puberty. The increase is most marked during mid and late puberty and
correlates best with pubertal stage, bone age, and time from peak height velocity
(PHV).
The maturation of bones is influenced by thyroid hormones, adrenal androgens, and
gonadal sex steroids, mainly estrogen. An excess secretion of these hormones can
lead to advanced bone maturation, and at the time of puberty, deficiency causes
delay.
At puberty, both sex steroids and growth hormone participate in the pubertal
growth spurt. The ending of the growth spurt is secondary to epiphyseal closure,
due to the action of the sex steroids.
An increase in physical size is a universally recognized event of puberty. Average
growth velocities decrease from the first year of life until puberty from 25 cm/year
during the first year of life to 5-6 cm/year during years 5 to 10. During puberty
height velocity increases and peaks during the adolescent growth spurt. Landmarks
include:
HEIGHT
The beginning of the increase in growth velocity is about age 11 in boys and 9 in
girls but varies widely from individual to individual.
The peak height velocity occurs at a mean of 13.5 years in boys and 11.5 years in
girls.
Pubertal growth accounts for about 20% of final adult height, a total averaging 2328 cm in females and 26-28 cm in males.
The average growth spurt lasts 24-36 months.

Growth during the year of PHV in the normal female averages 9 cm/yr and varies
normally from 5.4 cm to 11.2 cm. In the normal male, the PHV averages 10.3 cm/yr
and varies normally from 5.8 cm to 13.1 cm.
Males on average are 12-13 cm taller than females primarily because of the 2-year
delay in bone closure as compared to females. This accounts for about a 10-cm
difference between the two sexes; in addition, males also have 2-3 cm more of
growth during their growth spurt.
WEIGHT
Weight velocity increases and peaks during the adolescent growth spurt.
Pubertal weight gain accounts for about 50% of an individual's ideal adult body
weight.
The onset of accelerated weight gain and the peak weight velocity (PWV) attained
are highly variable. (Normal weight for age percentile curves are available through
the Centers for Disease Control and Prevention, 6525 Belcrest Road , Hyattsville ,
MD 20782-2003 . They can also be obtained on the CDC website at:
www.cdc.gov/growthcharts/ .
Differences in Growth Spurts between Males and Females
PHV occurs about 18-24 months earlier in the female than in the male.
PHV in females averages 2 cm/yr less than in males.
PWV coincides with PHV in males, but PWV occurs 6-9 months after PHV in females.
Prediction of Mature Height
While predicting adult height is a difficult task, individuals have used both the bone
age in calculations or a measure using midparental height as most individuals have
an adult height that is within 2 inches of the midparental height. This is calculated
using:
For girls:
(father's height - 13 cm) + mother's height
2
For boys:
(father's height + 13 cm) + mother's height
2
BODY COMPOSITION

BODY MASS INDEX


Body Mass Index (BMI) is an important measure for assessment of appropriate
weight for height
BMI is determined as follows:
Metric Formula:
Weight in kilograms height in meters height in meters = BMI
English Formula:
Weight in pounds by height in inches height in inches X 703 = BMI
The BMI declines until ages 4-6 years and gradually increases through adolescence
and adulthood. Children with an earlier increase in BMI are more likely to have
increased BMIs in adulthood. BMI tables can be obtained from the National Center
for Chronic Disease Prevention and Health Promotion ( www.cdc.gov/growthcharts/)
Lean Body Mass
Lean body masses decreases in females from about 80% to 75% at maturity while in
males it increases from about 80% to about 90% at maturity due to an increase in
skeletal muscle mass. In females the percentage of body fat increases.
Skeletal Mass
The increase in skeletal muscle mass during puberty is critical and peak bone mass
is achieved by early adulthood (the lifetime "bone bank"). Epiphyseal maturation
occurs under the influence of estradiol and testosterone. The assessment of this
skeletal maturation (bone age) is an excellent index of physiological maturation and
assessment of growth potential.
Assessment of sexual maturity ratings are listed below:

Males
o Genital stage 1 (G1) : Prepubertal
Testes: Volume less than 1.5 mL
Phallus: Childlike
o Genital Stage 2 (G2)
Testes: Volume 1.6-6 mL
Scrotum: Reddened, thinner, and larger
Phallus: No change
o Genital Stage 3 (G3)
Testes: Volume 6-12 mL
Scrotum: Greater enlargement
Phallus: Increased Length

Genital Stage 4 (G4)


Testes: Volume 12-20 mL
Scrotum: Further enlargement and darkening
Phallus: Increased length and circumference
o Genital stage 5 (G5)
Testes: Volume more than 20 mL
Scrotum and phallus: Adult
Females
o Breast stage 1 (B1)
Breast: Prepubertal; no glandular tissue
Areola and papilla: Areola conforms to general chest line
o Breast stage 2 (B2)
Breast: Breast bud; small amount of glandular tissue
Areola: Areola widens
o Breast stage 3 (B3)
Breast: Larger and more elevation; extends beyond areolar
parameter
Areola and papilla: Areola continues to enlarge but remains in
contour with the breast
o Breast stage 4 (B4)
Breast: Larger and more elevation
Areola and papilla: Areola and papilla form a mound projecting
from the breast contour
o Breast stage 5 (B5)
Breast: Adult (size variable)
Areola and papilla: Areola and breast in same plane, with papilla
projecting above areola
Male and female: pubic hair
o Pubic hair stage 1 (PHI)
None
o Public hair stage 2 (PH2)
Small amount of long, slightly pigmented, downy hair along the
base of the scrotum and phallus in the male or the labia majora
in females; vellus hair versus sexual type hair (PH3)
o Pubic hair stage 3 (PH3)
Moderate amount of more curly, pigmented, and coarser hair,
extending more laterally
o Pubic hair stage 4 (PH4)
Hair that resembles adult hair in coarseness and curliness but
does not extend to medial surface of thighs
o Pubic hair stage 5 (PH5)
Adult type and quantity, extending to medial surface of thigths
o

Sexual Maturity Ratings: female pubic hair stages

click for full-size image


MALE SEXUAL DEVELOPMENT
Male sexual development generally begins with the attainment of stage G2, at an
average age of 11.6 years (range 9.5-13.5 years). The first physical sign of puberty
in 98% of males is testicular enlargement. Ejaculation often occurs during SMR3
while SMR4 is often associated with fertility but may occur during SMR3. Puberty
takes about 3 years to complete but may range from 2 to 5 years. The typical
sequence of pubertal events in males is seen below which demonstrates the usual
late occurrence of peak height velocity at an average SMR of 4. The typical
sequence is adrenarche, beginning of growth spurt, testicular development,
beginning of pubic hair, peak height velocity
Sequence of pubertal events in males:

click for full-size image


FEMALE SEXUAL DEVELOPMENT
In most females, the beginning of a breast bud is the first physical sign of puberty.
While the traditional mean age of female sexual development was in the early 11s,
over the past decade in developed countries, this age has been decreasing. For
example, in the United States , the mean age of onset of breast development is
8.87 years for African-American girls and 9.96 years for white girls. The mean ages

for the onset of pubic hair are 8.78 years and 10.51 years, respectively. Potential
reasons for this decrease in age of onset, while unknown, might include improved
nutrition, increasing obesity, hormonal exposures and other environmental/societal
alterations. There may be important future consequences of earlier maturation with
regards to teen behavior, sexual activity and pregnancy as well as future lifetime
health consequences of early sexual maturation such as potential increase risk of
breast cancer.
During puberty, the female's breasts develop and the ovaries, uterus, vagina, labia,
and clitoris increase in size. The uterus and ovaries increase in size fivefold to
sevenfold. Completion of puberty in females averages 4 years but can range from
1.5 years to 8 years. In the average adolescent female, the growth spurt starts
about 1 year before breast development and this is followed by an average of 1.1
years until PHV and then followed in an average of 1 year by menarche. The typical
sequence of pubertal events in females is seen below which demonstrates the usual
early occurrence of peak height velocity at an average SMR of 2 and the late
occurrence of menarche at an average SMR of 4. Menarche occurs in 19% of
adolescents during PH3 and in 5 6% during PH4. There is little or no correlation
between adult height and either age of onset of growth spurt, age of PHV, velocity
at peak, or pubertal height gain. However, there is a correlation between adult
height and the height at onset of growth spurt or height at PHV.
Sequence of pubertal events in females

click for full-size image


There are wide variations of puberty between individuals and these are discussed in
B1.
back to top
Questions
Question #1
What is the first visible sign of puberty in males?

Answer #1
The first visible sign of puberty in males is an increase in testicular size. However,
an increase in height velocity occurs before this although this is unlikely to be
noticed in routine growth chart.

Question #2
What is the first visible sign of puberty in females?
Answer #2
Increase in breast development.

Question #3
The following male is about what stage of pubic hair?

click for full-size image


Answer #3
Stage 3 since there is a scant amount of ADULT hair. In stage II, there is sparse
amount of fine non-adult hair. In stage four seen below there is normal adult hair
but not quite out to medial thigh. Review of sexual maturity ratings.

click for full-size image

Question #4
What breast stage is indicated below?

click for full-size image


Answer #4
Stage 4. In stage two there is small breast bud and in stage 3 there is larger and
more elevation of breast and areola enlarges but remains in contour with breast. In
the photo above, there is a secondary mound and this places this teen in stage four.
In stage 5, which some females never attain, the areola flattens out

Question #5
Peak height velocity is an early or late event in most females?
Answer #5
In males PHV is usually a late pubertal event at about an SMR of 4 but in females,
PHV is usually occurs earlier during puberty. In females PHV is most common during
SMR 2 while menarche occurs in about 2/3rds of females at about SMR 4.

Question #6
Most of the difference between the height of males and females is because the PHV
in males is greater? True or false?
Answer #6
False: About 80% of the difference between male and female height is the later age
of PHV in males which allows for greater growth before epiphyseal closure. The
difference in amount of PHV between males and females is probably responsible for
about 20% of the difference.

Question #7
In developed countries, it appears that there has been a reversal of the earlier onset

of female puberty and so the average age of the start of sexual development has
increased by about 6 months? True of false?
Answer #7
False: In developed countries the onset of female sexual development has
continued to decrease in age.
back to top
Cases
Case #1
A fourteen-year-old adolescent male comes to see you in the office. The teen
complains of fatigue for about six months with no other symptoms. His psychosocial
profile and history is normal. His examination is normal with no focal abnormal
signs. You order a CBC and his hemoglobin is 12.0 gm/DL.
Is this teen anemic and do you have an explanation for his fatigue.
Answer:
This is a bit of a "trick" question as you really cannot answer the question without
first knowing the sexual maturity rating of the adolescent.

click for full-size image


The three teens above are all fourteen years of age. The one on the left is about a
SMR of 2, the middle about a 3 and the one on the right a 4 to 5. All three of these
teens are normal for age 14. However, there is a significant correlation between
SMR and hemoglobin/hematocrit levels in pubertal males. In males, with increasing
SMR, there is an increase in testosterone and thus an increase in erythropoietin
levels. The rising erythropoietin levels increase both skeletal muscle mass and also
increase hemoglobin production. Thus, a normal hemoglobin level for the teen on
the far left above might be in the 12.0 - 14.0 gm/dl while a normal level for the teen
on the right might be in the 14.0 -18.0 gm/dl level. A hemoglobin level of 12.0
would be normal for a teen in early development but anemic for a teen in near adult
development. In conclusion, for the teen on the left, the blood count cannot explain
the teen's fatigue while for the teen on the right, it is an indication of some
abnormality that needs further evaluation.

Case #2:
Your next teen in the morning clinic gives you a history of feeling well but is being
referred from an outside clinic for an abnormal blood test. His history is completely
negative as well as his psychosocial history. The teen comes in with laboratory from
an outside physician that shows a normal CBC and normal chemistry panel with the
exception of an alkaline phosphatase that is about 50% above normal. On
examination, there are no abnormal findings and the teen is a SMR of four. Are you
concerned?
Answer:
This is a common occurrence in developing adolescents. With rapid bone growth,
alkaline phosphatase levels increase and one can find a level that might be 50100% above the normal range. This tends to peak at about the peak height velocity
in males and females so occurs earlier in females. The graph below demonstrates
this relationship between alkaline phosphatase levels and SMR in both males and
females.

click for full-size image


How would you expect this teens body composition (body fat, skeletal muscle mass)
to compare to a female also age 14 and SMR of 4.
Of note is that males and females have opposite affects of puberty on muscle mass
and body fat. In males, under the influence of testosterone, skeletal muscle mass
increases and percent body fat decreases as seen in the graph below. In a female,
under the influence of testosterone, body fat increases. Thus in this teen one might
expect a per cent body fat of about 11% in a healthy male and over 20% in a
healthy female of SMR 4.

click for full-size image


Interviewing and communicating with adolescents
Interviewing and communicating
" The style and personality of the practitioner and his/her philosophy of medical
care are considered to be most important in the medical care of adolescents. The
practitioner should be mature and open-minded. He/she should be genuinely
interested in teenagers as persons first, then in their problems, and also in their
parents. He/she should not only like teenagers but must also feel at ease with them.
He/she should be able to communicate well with his/her patients and their parents.
The practitioner should help to enhance family communication while assuring
confidentiality when requested around personal issues."
( Adapted from: Committee on Care of Adolescents in Private Practice of the Society
for Adolescent Medicine).
GENERAL GUIDELINES FOR THE OFFICE VISIT
There are a few important guidelines in working with teens:
Liking the Adolescent - Important for the clinician working with adolescent to like
adolescents. If the clinician has an aversion to adolescents and their problems, it is
likely best to refer this age group to another colleague.
Involving the family - The family is a critical component in the care of an
adolescent and it is important for the clinician to introduce himself or herself to the
family. It is also important to spend time discussing the concerns of the parents.
While more of the visit may be spent with the adolescent alone, it is important for
the parents, in most cases, to be included at some point in the visit. This might be
at the beginning, end or both depending on the age of the adolescent and the
complexity of the problem. At the end of the visit, the clinician should summarize
the findings and plan with the teen and if the parents or guardians are involved,
summarize issues that can or must be discussed with family members. Although the
adolescent may be the primary patient, the parents cannot be overlooked. Parents'
input and insight are crucial, for in a real sense the family is the patient.
It is also important to consider that the definition of a family has changed and there
may be many possible family constellations including blended families, stepfamilies,

adoptive families and foster families. Family cultural and ethnic backgrounds are
also critical to helping to understand the teen and their family.
Establishing rapport - It is important but not always easy to establish rapport with
an adolescent during the first visit or several visits. Helpful suggestions include:

Introducing yourself to the teen and parents or guardians.


Chatting for brief period about the teens outside activities including hobbies
or school.
Letting the teen talk for awhile on topics or areas they feel like talking about.
Treating the adolescent's comments seriously
Moving from less threatening health subjects such as review of systems to
more difficult topics such as sexuality and drugs.
Exploring the issues that concern the teen - not only those concerns of the
parents.

Ensuring confidentiality - It is critical to insure a sense of confidentiality with the


teen. In this regard the health care practitioner should be familiar with those laws
and regulations that cover consent and confidentiality among minors in their
particular country, state, province or other locality. The limits of confidentiality
should also be discussed. Parents should also be aware of these confidentiality
guidelines.
Acting as an advocate - Since the adolescent may have had encounters with
some adults who have been non-supportive, this is an opportunity for the clinician
to stress the teen's positive attributes, characteristics and abilities. This is not the
same as supporting high-risk behaviors.
Listening and displaying interest - Listening closely to the teen can be a key to
developing rapport. This can include being cautious in giving advice when asked,
trying to understand the teen's perspective and staying focused on what the teen is
telling you. Demonstrating concern and interest is also helpful in establishing
rapport.
Discovering the hidden agenda - It is very common for an adolescent to present
with a complaint that does not represent the major issues that the teen is
concerned about. It is also common that parents may present concerns that are not
the major issue for the teen. For example, a teen may come in complaining of a
headache or acne, but is really concerned about being pregnant or having a
sexually transmitted infection. It is critical for the clinician to be aware of these
other issues that may be more threatening to the teen's health then their chief
complaint. A review of the HEADSS assessment below can help elicit this
information.
Using a developmentally oriented approach - While it is important to cover
areas of sex, family, peer group, and drug use, the clinician must keep in mind the
developmental state of the adolescent. A 12 year old pre-pubertal male would not
be asked the same questions in the same manner as would be asked a 18 year old
fully mature male.

Information gathering - There are several methods that might be used to elicit
both health information and psychosocial information. Traditionally this is through
one and one interviews. Another method is a health assessment form. Examples for
adolescents from the AMA Guidelines for Adolescent Preventive Services (GAPS) are
at http://www.ama-assn.org/ama/pub/physician-resources/public-health/promotinghealthy-lifestyles/adolescent-health/guidelines-adolescent-preventive-services.shtml
. There has been a growing interest in using computerized techniques to help assess
health status in both teens and adults. In some studies, this may even be preferred
by many teens. One approach that was developed at Childrens Hospital of Los
Angeles is to obtain psychosocial information using the HEADSS interview. This
includes the topics of Home, Education, Activities, Drugs, Sex (activity, orientation,
and sexual abuse), and Suicide. This includes questions such as:
Home Where is the teen living? Who lives with the teen? How is the teen getting
along with parents and siblings?
Education Is the teen in school? What classes is he or she doing well in? What
goals does the teen have when he or she finishes school? If the teen is older out of
school, the practitioner should ask about employment.
Activities What does the teen do after school? What does the teen do to have fun
and with whom? Does the teen participate in any sports activities? Community or
Church activities? What are the teen's hobbies? This may be an opportunity to
explore issues of seat belt safety or bicycle helmet safety.
It is useful to reassure confidentiality again before questions about drugs and
sexuality.
Drugs What types of drugs are used by the teen's peers or family members use?
What types of drugs does the teen use and what amount and frequency and is there
intravenous use? This includes both alcohol and tobacco. It can be useful to begin
questioning with a less invasive approach such as: "I know that drugs are fairly
common on school campuses. What drugs are common on your campus?" and "It is
not uncommon for some teens to try some of these drugs. Have any of your friends
tried them? and "How do you handle the situation when your friends are using
drugs? Do you ever try?
Sexuality Is the teen dating and what are the degree and types of sexual
experience? Is the teen involved with another individual in a sexual relationship?
Does the teen prefer sex with the same, opposite, or both sex (es)? Has the teen
had sexual intercourse? This is also to find out how many partners the teen may
have and also a history of both sexually transmitted infections and contraceptive
use.
An approach might be to ask something like: "Laurie, I mentioned that I might be
asking some questions that were personal but very important to your health. Again,
this is information that I will be keeping confidential. The area I want to discuss has
to do with relationships. Are you going out with anyone right now?" and something
like: " As you know, there are many teens who are sexually active. By that I mean

that they have had sexual intercourse. There are also many teens who have chosen
not to have sexual intercourse. How have you handled this part of your relationship
with Bill or with other boys you have dated?
Suicide Has the teen had any prior suicide attempts? Does the teen have any
current suicidal ideation?
Sexual Abuse or Physical Abuse These can be critical areas to ask about
particularly in adolescents with any significant problems in the areas listed above
such as family dysfunction, change in school grades, lack of friends, substance
abuse, early onset of sexual activity, history of suicide attempts or runaway
behavior.
Interview tips: Help interview tips with adolescents include:

Shaking hands with the adolescent first.


Avoiding lecturing and admonishing.
Focusing on the initial history taking on the presenting complaints/problems.
Having a positive attitude towards the adolescent
Avoiding judgmental responses - taking a neutral stance
Avoiding medical jargon
Being attentive, genuine and empathic
Identifying who has the problem (i.e., is this problem the teen's concern or
the parents').
Avoiding writing during the interview, especially during sensitive questions.
Criticizing the activity, not the adolescent and highlighting the positive.

Physical examination : The physical examination may provide another


opportunity to teach the adolescent about their changing body. Reassurance about
normal findings may also be helpful. Sometimes the true chief complaint is
disclosed during the examination.
Closure: When the history and physical assessment are complete, the clinician
should give the teen a brief summary of the proposed diagnosis and treatment.
Issues that should be discussed with the family should also be addressed at this
time. Also at this time resources should be discussed and a follow-up appointment
made as needed. The adolescent should also have time to ask final questions.
OFFICE SETUP
The space that adolescents are seen for their care can also be helpful in their
overall care.
Space: Adolescents prefer not to be treated as children and the more private their
space and waiting area the better. Materials in the waiting area and clinical offices
appropriate for their age group is helpful. The examination table should not face the
door and curtain should be available for privacy. If possible the desk in the office

should be oriented so that the health-care provider sits beside the desk, not behind
it.
Appointments : Time can be a problem with the adolescent visit particularly for
the first visit. More time should be allotted for this visit to allow for discussing their
past medical and psychosocial history. If the clinician is pressed for time, doing the
history at the first visit and the physical examination on another day is a reasonable
approach.
Billing : In regions where teens may be required to pay for their visit or the parents
will receive a bill, arrangements should be discussed early. Confidentiality can
become a problem in certain billing situations and may require special
arrangements. The adolescent must realize that an insurance payment may result
in parents finding out about visits and the diagnosis; however, a neutral diagnosis
can be used in most situations.
Availability of educational materials : It is helpful to place books, pamphlets,
hot line numbers and reliable web site information in the waiting room or office on
topics such as puberty, sexually transmitted diseases, sexuality, and contraception.
Note taking: The practitioner should take as few notes as possible during the
interview.
PARENTS
Often parents come to the health care professional with requests for help with
parenting their teens. Helpful suggestions include:
Guidelines for parenting

Listening to the teenager


Treating his or her comments seriously and avoid minimizing a problem.
Being flexible
Avoiding power struggles
Showing interest in the teen and their activities
Spending time together both working together and having fun together
Showing trust in the adolescent
Avoiding comparison with other teenagers
Avoiding lecturing or moralizing
Avoiding overreacting, especially reaching conclusions based only on
appearance, dress, or language.
Avoiding phrases such as: "The trouble with you is...." or "How could you do
this to me?" or "Is that all? I thought it was something important." Or "in my
day" or "That's a dumb thing to say"
Stressing positive attributes of the adolescent.
Respecting each other's privacy
Keeping a sense of humor

Resolving conflicts together. Decisions that occur in the home about the
adolescent should involve the adolescent's input and may involve the whole
family.

House Rules: House rules may help a family work together better. These include
the expectations for behaviors for the family to live together as a group. It is helpful
to have these rules worked out with input from the whole family and for them to be
written down. The rules should be fair and consistent with associated consequences
if the rule is broken. Teens may be eager to participate in the establishment of such
rules when they find out that they might include a rule such as "no one will enter
someone else's room without knocking first." Rules are mainly needed for teen or
family member behaviors that are a problem and there should be a maximum of 5 10 rules. Some examples include:

Dinner will be at about 6 PM and everyone is expected to be home and ready


to eat at that time.
Family members are expected to speak courteously to each other.
Before opening someone's door, knock and wait for an answer.

Other parenting issues include:


As teen's peers become an increasingly important influence and the teen
seeks more independence, parents must adapt to change in relationship with
their teen.
Experimentation by teens: Important for parents to remember that while teens
may experiment with many types of behaviors, most teens accept their parent's
basic values. Parents can set firm, fair and explicit limits around teens behavior.
Parents must not overreact to rejection of one or both parents by the teen for a
time period.
A dolescents are at maximal growth velocity and change and may be more
vulnerable to social risks such as drugs, risks of parts of sexuality, domestic
violence, and poverty.
Modern family issues such as less intact families and less extended families add
additional challenges.
The violent messages added through the media add to the challenges.
Invulnerability: Adolescents feelings of invulnerability also add to the risks that
teens place themselves.
Questions
Question #1

Suggest a few helpful concepts during the first visit with an adolescent and their
family.
Answer #1
Some helpful concepts during the first visit include:

Assuring confidentiality

Involving the family unless there are particular contraindications

Using good listening techniques

Being aware of the hidden agenda

Interviewing the adolescent with a developmentally oriented approach

Including a psychosocial history in the history taking

Making sure the adolescent understands the diagnosis and treatment


plan.

Liking the Adolescent

Question #2
What changes could the clinician make to a clinical setting to make it more
"adolescent friendly"
Answer #2
Suggestions might include:

Having appropriate materials in the waiting room and offices for teens

Setting up special times for teens to come to the clinic

Making sure the exam table does not face the door and have privacy
curtains available

Allowing more time for the first visit

Discussing billing arrangements with teen and parents if billing and


confidentiality issues arise

Question #3

What items are critical to ask in the psychosocial history of a 16-year-old


adolescent?
Answer #3

Home situation

Educational/school issues

Activities and hobbies the teen is involved in

Drug use including alcohol, cigarettes and other drug use

Sexuality issues including relationships, types of sexual activity, sexual


orientation, contraception, STIs.

Suicidality and mental health assessment

Sexual or physical abuse particularly in teens with higher risk profile or


problems

Question #4
You are about to examine a 12-year-old girl who has complaints about breast lumps?
Who should be in the examination room?
Answer #4
Certainly a male examiner should have a female chaperone in the exam room. It
might be important to ask the teen if she prefers to have her mother present. Some
younger teens prefer to a parent present while others do not. It would be less
appropriate to have a parent present for an older adolescent.
Cases
Case #1
Part I:
You are scheduled to see a sixteen-year-old adolescent female named Leslie who
has recently been complaining of headaches and abdominal pain. She is in the
waiting room with her mother.
Question:

How would you first approach seeing this combination of adolescent and her
mother?
Answer:
There are probably a couple of ways to approach this teen and her mother. It would
be important to introduce yourself to both the mother and the teen. The first
approach might be to see the daughter first and take the appropriate history and
then have the mother come in to see what her concerns might be and how she
interacts with her daughter. Then one could see the daughter without the mother
for the physical examination. An alternative approach might be to see them
together to see how they interact and after obtaining some of the mom's concerns
and a short family history one could excuse the mother and interview the
adolescent. In either case, at the end of the examination, it would be important to
sum up the information for the teen and then bring in the mother to convey
information that is not confidential.
Part II
You see the teen and the mom together first and the mom does not let the teen
really answer or give much information. The mother is concerned that the teen may
have some kind of tumor. She mentions that the teen has been extremely difficult in
recent months and does not listen to her or her husband about when to be home
and how much she should be studying. She states that she seems to spend a lot of
time with some guy named Tom who she does not really like. You thank the mom for
her concerns, that they are important and you will discuss these with her daughter.
You also explain that you will be spending some time interviewing and examining
the teen alone. You explain the importance of spending time with the adolescent
alone as she is a developing adult.
Question:
What information would be particularly important to obtain from this adolescent as
part of the history?
Answer:
Medical history - This would information about what concerns the teen has and in
particular a about her headaches and abdominal pain.

She discusses that the headaches are not very severe and that
she has had occasional headaches when she is stressed for
about 5 years. They are more frequent when she has school
exams or she is fighting with her parents over her friends. She
has no associated neurologic symptoms and the headaches
usually resolve with ibuprofen or over a couple of hours. There
has been no increase in severity

The abdominal pain also has been very mild and is associated
with stress. They are not related to eating or bowel movements
and there are no other associated gastrointestinal complaints.
The pain is midline without radiation. The teen states she is not
very concerned about the pain.

Sexual history : After reassuring the teen that information about her sexuality will
be confidential, you ask about her relationship with Tom and other individuals. She
discloses that Tom is her first boyfriend and that they have been having sexual
intercourse for six months. He uses condoms occasionally. She states that she
thinks she cannot get pregnant because her periods have always been irregular, but
she has been more concerned recently because she has stopped having periods for
over two months. She uses no other contraception. She has no history of any
vaginal discharge or genital lesions or history of STIs. She has not had sex in at
least two weeks.
Menstrual history : She had her first menses at age 12.5. They have always been
somewhat irregular and occur about once every two months. Her last menses was
over two months ago. She occasionally has cramps with her menses.
Home situation: She lives with her mother, father and one brother. They usually get
along but recently when she has been going out with Tom she has felt her parents
have been very angry with her for going out and distrustful of where she is and
what she is doing. She does not talk much with her father.
Drug history: She denies any drugs except for an occasional beer on the weekends
and trying marijuana a few times.
Mental health: She states that she is usually fairly happy but she is concerned about
the possibility of pregnancy and she is worried that if her parents found out they
"would kill me". She mentions that the headaches and abdominal pain got worse
when she started worrying about being pregnant. She has no suicidal ideation and
has never been physically or sexually abused by anyone.
Part III
You perform a physical examination. Her vital signs are normal. Her general
examination is unremarkable. Her abdominal examination shows no organomegaly
and no tenderness. Her neurologic examination is also normal. You explain the
importance of a pelvic examination and what is involved. A pelvic examination
shows no genital lesions, no vaginal discharge, normal cervix and no adnexal or
uterine tenderness. You also perform a Pap smear, gonorrhea and chlamydia test.
Question:
What might you wish to discuss with the teen at this point?
Answer

Possibility of pregnancy - This might be a good time to review with the teen issues
of the possibility of pregnancy and how she might approach this if she had a
positive pregnancy test. This would include would she involve her parents, if not
why not; had she thought about options if she were pregnant; and possibilities of
contraception if she were not pregnant at that point. You let her know that you will
be ordering a pregnancy test at this point and will review those findings first with
her only.
Summary of her physical findings: You reassure the teen that her history and exam
do not suggest any serious problems in regards to her headaches and abdominal
symptoms. It is quite possible that they are related to the stress that she is under
recently.
As she has not had a blood test in at least ten years you order a CBC and urine
pregnancy test.
The pregnancy test is negative and the CBC is ordered. You discuss with the teen
the results of the pregnancy test and while she is reassured that she is not
pregnant, you point out that she still could become pregnant. You also point out that
while the condoms are a great idea to protect for STIs, she should be considering
alternatives for additional protection against pregnancy. She has previously thought
about oral contraceptives, wants to start and wants to know if she can do this
without discussing this with her mother.
Question:
Can you prescribe oral contraceptives to this adolescent without her mother's
knowledge or consent?
Answer:
The answer to this question depends on the laws and regulations in your own region
or country. In many areas, regulations allow minors to consent for prevention of
pregnancy including contraception. It is important to know the regulations for the
area that you practice in. In fact in many areas, not only do minors have the right to
consent but that is associated with the right to confidentiality and privacy over this
information, so one would not have the right to disclose this information to a parent.
You discuss the options with the teen, you discuss the possibility of talking with the
mother with you acting as a mediator with the mom. However, she still refuses to
have the mom involved with her decision. You review contraceptive options and she
chooses the birth control pill. You prescribe her an oral contraceptive pill.
Question
What are important issues to discuss with the teen and the mother together?
Answer

Review of the results of her history and examination. At this point you review with
the mom and the teen that her history and examination suggest tension headaches
and not a tumor and that both the headaches and her abdominal pains probably
relate to stress from both school. They also may relate to some of the tension
regarding the disagreements that they have over her relationship with Tom.
Discussion with mom and teen about relationship with Tom: It would be important to
explore Mom's concerns about the relationship. It would also be important at this
point to assess how dysfunctional you think the relationship is between Leslie and
her parents and whether you could intervene yourself with one or several follow-ups
or whether at this point a referral to a counselor would be appropriate.
Confidentiality Issues
The rights of minors and in particular adolescents can be confusing. Adolescents are
individuals who have more mental capacity for decision making than younger
children but are not yet full adults. There are many specific areas regarding consent
and confidentiality that are particularly difficult for teens, parents, health care
professionals and lawmakers. These usually surround areas of reproductive health,
mental health and substance abuse. There are also significant differences between
countries and individual states or provinces within countries regarding particular
laws of adolescent rights to consent and confidentiality.
Over the last several decades the legal framework that applies to the delivery of
adolescent health care has changed in several ways.

The United Nations has enacted the UN Convention on the Rights of the Child
(http://www.unicef.org/crc/index_30160.html or see summary below)
Courts have recognized that minors, as well as adults, have constitutional
rights.
All states in the U.S. have enacted statutes to authorize minors to give their
own consent for health care in specific circumstances.
The financing of health care services for all age groups and income levels has
undergone major change

In the United States , the rights of adolescents took a major step with Gault in 1967,
in which the United States Supreme Court stated that "neither the Fourteenth
Amendment nor the Due Process Clause is for adults alone." However, most specific
legal provision that that affect adolescents' access to health care are contained in
state and federal statutes or in common law decisions of the courts.
It becomes essential that health-care practitioners treating adolescents have a clear
understanding of the legal framework within their particular country or state
including checking:
In most states and countries, children under 18 have legal status that differs from
that of adults. Several areas are of particular concern. These include:
CONSENT

Who is authorized to give consent for health care and whose consent is required?
In general, U.S. law requires the consent of a parent before medical care can be
provided to a minor. However, there are numerous exceptions to this requirement.
These may include:

Consent by someone other than a biologic parent - such as a foster parent, a


juvenile court, a social worker, or probation officer
Emergency situations where care may be provided without prior consent to
safeguard the life and health of the minor.
Specific legal provisions in particular states that allow minors to consent for
specific areas of care. Some of these include
Contraceptive care
Pregnancy related care
Diagnosis and treatment for sexually transmitted diseases (STDs)
Diagnosis and treatment of either human immunodeficiency virus (HIV), or
acquired immunodeficiency syndrome (AIDS).
Diagnosis and treatment of reportable or contagious diseases
Examination and treatment related to sexual assault
Counseling and treatment for drug or alcohol problems
Counseling and treatment for mental health issues.

In addition, many states have given consent rights to minors who have special
status. These include:

emancipated minors

married minors

minors in the armed services

minors living apart from their parents

and in some states "mature minors"

Not all states have statutes covering all of these services. Some of these statutes
contain age limits, which most frequently fall between ages 12 and age 15 years. A
state by state analysis is available at:
http://www.guttmacher.org/graphics/gr030406_f1.html As theses vary from country
to country and state to state, clinicians are advised to check laws in their own area.
Informed consent describes the process during which the patient learns the risks
and benefits of alternative approaches to management and authorizes a course of
action proposed by the clinician. Informed consent has both ethical and legal
derivations. Informed consent also implies that the individual has the mental
capacity to given informed consent.

Assent: Under specific legal circumstances, adolescents may receive confidential


care and may give informed consent for recommended care. If the legal
circumstances do not allow a minor to consent for medical treatment, the minors
views and opinions can still be respected by obtaining assent. This respects the
decision-making skills of a minor by allowing them to participate in the decision.
PRIVACY AND CONFIDENTIALITY
Aside from consent, there is also the issue of confidentiality of services. This
includes who has the right to control the release of confidential information about
the health care, including medical records, and who has the right to receive such
information?
There are numerous reasons why it is important to maintain confidentiality in the
delivery of health-care services to adolescents. These include:

The needs of clinical practice: Confidentiality is often needed to facilitate


adolescents seeking necessary care and also in providing accurate, candid
and complete health information.
Developmental Needs: Confidential discussions and disclosure help support
the adolescents' growing sense of privacy and autonomy.
Safety Issues : There are also times that confidentiality/consent is important
to protect teen from humiliation and discrimination that could result from
disclosure of confidential information.

There are numerous country and local regulations that can affect this confidentiality.
Because of the potential for many conflicting regulations, clinicians are advised to
check on local regulations that apply to confidentiality with minors. It is important to
check out:

What information is confidential (since it is confidential information that is


protected against disclosure)?
What information is not confidential (since such information is not protected)?
What exceptions are there in the confidentiality requirements?
What information can be released with consent?
What other mechanisms allow for discretionary disclosure?
What mandates exist for reporting or disclosing confidential information?

Legal Limits of Confidentiality


It is important to balance the moral needs of protecting the rights of the adolescent
with the legal and ethical obligations to breach this confidentiality in selected
instances. There are circumstances in which it is neither possible nor appropriate to
maintain the confidentiality of information for legal and other reasons. These
include situations in which the adolescent poses a severe risk of harm to himself or
herself or to others, and cases of suspected physical or sexual abuse for which there
is a legal reporting requirement. There are also specific laws in some geographic
areas that require parental notification in certain circumstances, even if the care is

based on a teen's own consent. Finally, when confidentiality must be breached for
ethical or legal reasons, the adolescent should be informed.
Medical Records
Confidentiality protections apply not only to verbal communications but also to
written information contained in medical records. Patients, who are permitted to
consent to their own health care, should be allowed to review their own medical
records and to protect their medical records from review by others. It is far more
difficult to protect the confidentiality of written medical records.
It is important to understand local regulations regarding the release of medical
records of adolescents. One should understand that many or most hospitals and
clinics will release minors medical written chart information to parents with parental
consent without requiring the permission of the minor adolescent. This may break
the confidentiality of information with an adolescent.
Although usually bound together in clinical encounters, confidentiality and consent
are different. Confidentiality can occur during an encounter whether or not specific
informed consent for a treatment or intervention is given. For example, pregnancy
options may be confidentially discussed before informed consent is given for a
pregnancy intervention.
PAYMENT
A last issue that arises with consent and confidentiality is occasionally that of
payment of services. Who is financially liable for payment and is there a source of
insurance coverage or public funding available that the adolescent can access. The
fact that a minor has the right to consent and confidentiality of services does not
necessarily guarantee payment, nor confidentiality of the information if insurance is
used. In addition, some consent laws specify that if a minor is authorized to consent
to care, it is the minor rather than the parent who is responsible for payment.
A source of payment is essential whether an adolescent needs care on a
confidential basis or not. Adolescents are uninsured and underinsured at higher
rates than other groups in the population and those adolescents living below the
poverty level are at the greatest risk for lacking health insurance. This can present a
significant barrier to care.
IMPORTANT DOCUMENTS REGARDING MINORS' RIGHTS
http://www.unicef.org/crc/index_30160.html
Overall the UN Convention:

Reinforces fundamental human dignity


Highlights and defends the family's role in children's lives
Seeks respect for children but not at the expense of the human rights or
responsibilities of others
Endorses the principle of non-discrimination .
Establishes clear obligations

If a decision is being made by any organisation about a child or youth, then


their interests must be considered when making the final decision.

Article 4: Governments have made a commitment to live up to the Convention's


standards
Article 5: Governments must value and support parents and other adults in their
roles as carers. Parents and others have a responsibility to listen to children/youth
and vice-versa.
Article 6: Children have the right to life and must have the best possible chance to
develop fully
Article 7: Every child has the right to a name at birth and the right to become a
citizen of a country.
Article 8: Governments must be committed to respect children's right's to preserve
their nationality and identity
Article 9: Children can only be separated from parents if it is in their own best
interests and if that happens, then someone who is an interested party must be
given the opportunity to take part in proceedings and have their views heard.
Article 10: States shall act quickly and in a positive and humane manner in
applications by families for reunification.
Article 11: As they mature, children have the right to freedom of thought and
religion.
Article 15: Children have a right to join organisations and to meet with each other.
They can also take part in meetings and peaceful gatherings.
Article 16: Children have the right to their own privacy.
Article 17: Both parents have the main responsibilities for bringing up their
children but governments are expected to recognise that some parents may need
help to care properly for their children if they are both working.
Article 19: Children must be kept safe from violence and they must be kept safe
from harm.
Article 20: If children cannot live with their family, they must be properly looked
after in some other way, for example, by another family or in a children's home. The
child's religion, race, culture and language must all be considered when a new home
is being chosen for the child.
Article 21: Children being adopted must only be adopted under very strict rules
which ensure that what is happening is in their best interests.

Article 22: States shall take appropriate measures to ensure that children who are
seeking refugee status or who are refugees shall receive appropriate protection and
humanitarian assistance.
Article 23: Governments shall recognise that a mentally or physically disable child
should enjoy a full and decent life.
Article 24: Children have the right to be as healthy as possible. If they are ill, they
must be given good health care to enable them to become well again. The
Government must try to reduce the number of deaths in childhood and to make sure
that women having babies are given good medical care.
Article 25: If a child is cared for by a local authority, the authority must review the
children's situation regularly.
Article 26: Governments should recognise that children have the right to benefit
from social security type of benefits.
Article 27: Every child has the right to expect an adequate standard of living. The
Government shall help parents to achieve this for their children.
Article 28: Every child has the right to free education at primary school level.
Different kinds of secondary school education should be available for children. For
those with ability, higher education should also be provided .
Article 29: Schools should help children develop their skills and personality fully,
teach them about their own and other people's rights and prepare them for adult
life.
Article 30: Children have the right to access their own culture, use their own
language and practice their own religion.
Article 31: Every child is entitled to rest and play and to have the chance to join in
a wide range of activities.
Article 32: The Government shall protect children from doing work which could be
dangerous or which could harm their health or interferes with their education.
Article 33: The Government shall take measures to protect children from
dangerous drugs.
Article 34: The Government shall protect children from sexual abuse.
Article 35: The Government shall take measures to protect children from being
abducted or sold.
Article 36: Children shall be protected from all sorts of exploitation which can
damage their welfare

Article 37: No child shall be subject to torture or inhumane treatment or


punishment.
Article 38: The Government should respect and ensure respect for rules of
international humanitarian law applicable to children during armed conflicts. No
child under 15 can be enlisted into an army.
Article 39: The Government shall promote physical and psychological recovery and
social reintegration for victims of neglect, abuse or torture.
Article 40: Children who have committed a crime, or who are alleged to have
committed a crime, should be shown respect for their human rights by those who
are dealing with them. They should have access to appropriate help including legal
assistance.
Article 41: If a country's own law better meets the rights of the child than the
Convention does, then the terms of the Convention will not apply.
The Government must publicise the Convention to parents and young people
throughout their country.
European Convention on Human Rights
http://www.hri.org/docs/ECHR50.html
Position Paper on Confidential Health Care: Society for Adolescent
Medicine
Journal of Adolescent Health 1997;21:4008-415
http://www.adolescenthealth.org/AM/Template.cfm?
Section=Position_Papers&Template=/CM/ContentDisplay.cfm&ContentID=2597
Highlights of position paper:

Health providers should inform adolescent patients and their parents, if


available about the requirements of confidentiality, including a full
explanation of what confidential care entails and the conditions under which
confidentiality might be breached.
Health providers must remain flexible when delivering confidential care to
adolescents. Blind adherence to absolute confidentiality, or absence of
confidentiality (in deference to parental wishes), is neither desirable nor
required by ethics or law.
Health providers should develop a disclosure plan for those adolescents who
are deemed not to have capacity to give informed consent or for whom
disclosure of information to responsible adults becomes necessary which
involves adolescent wishes about the manner in which information is shared.
Confidentiality considerations regarding record keeping are necessary. Health
providers must consider the manner in which written and electronic medical
records might be available to parties in ways that verbal communication are
not, and in ways that would be objectionable to adolescent patients.

Expanded efforts are needed to increase the education of health


professionals regarding the laws and regulations in their jurisdiction relating
to confidentiality and informed consent for adolescents. In addition, specific
training is needed to increase providers' skills in effectively and appropriately
incorporating confidentiality into clinical practice.

American Academy of Pediatrics Policy on Confidentiality in Adolescent


Health Care (RRE9151)
Key points in this policy include:

Clinicians should make every reasonable effort to encourage the adolescent


to involve parents
The adolescent will have an opportunity for examination and counselling
apart from parents
Confidentiality will be preserved between the adolescent patient and the
provider as between the parent/adult and the provider.
The adolescent must understand under what circumstances (e.g., lifethreatening emergency), the provider will abrogate this confidentiality.

Confidentiality of Health Care: Canadian Paediatric Society - Adolescent


Medicine Committee
Case
16-year-old female comes into the clinic with her mother for evaluation of acne and
headaches. When you are taking the history, she discloses that she has a boyfriend,
she is sexually active and he occasionally uses condoms. Her last menstrual period
was two months ago. She is concerned about being pregnant. She would like to get
a pregnancy test. She also would like to get contraceptive pills if she is not
pregnant. She asks that you do not tell her mother that she is sexually active or that
she is getting a pregnancy test.
Questions
Can you do a pregnancy test without the mother's permission?
Can you do a pelvic examination without the mother's permission?
Answers
These questions get to the heart of consent and confidentiality issues with teens.
Often the major concerns surround issues of reproductive health as in this teen. The
exact rules on consent and confidentiality depend on your own country and state's
regulations regarding consent and confidentiality among minors.
However, regardless, it would be very important to explore in this adolescent
several issues first.

What are her reasons for not wanting to disclose and share this
information with her parents? In some cases, after a discussion of the
issues, teens are willing to share information with their parents.

What is the nature of her relationship with her boyfriend and is it


consensual

Does she have any symptoms of a sexually transmitted disease?

If she is pregnant, has she thought about what she might do and who
she might share that information with?

Regarding the pelvic examination: This would be included in the overall


consent by parents for treatment if a general consent was signed. In
addition, in areas where minors can consent for treatment of
pregnancy, prevention of pregnancy or diagnosis of STDs, the minor
could give their own consent. However, often the logistics of doing a
pelvic examination with the mother in the waiting room can be difficult
and may entail another visit.

Many countries and local states or provinces allow for confidential care of
pregnancy in teens. This often includes both consent and the confidentiality of that
information.
The teen states that she has difficult relationship with her parents. She is very
concerned about what they would do if they found out about her sexual activity.
After discussing this for a while, she is clear that she wants this information
confidential. You are practicing in an area that allows for consent and confidentiality
of this health care and you reassure her about the confidentiality of this information
and limitations on confidentiality. She declines a pelvic examination at this time and
would like to come back in a couple of weeks. You complete your history and
physical examination. You also perform a urine pregnancy test and the test is
negative. You discuss oral contraceptive options with the adolescent. You also
discuss management of her acne and headaches. You bring her mother in for a
discussion of the patient's health care.
Question
Can you keep the information about the pregnancy test confidential and can you
prescribe OCPs without parental involvement?
Answer
Again, this depends on local regulations. However, in many areas, minors have the
right to contraceptive care or prevention of pregnancy without parental consent or
involvement. Usually the right of confidentiality follows the right of consent but not
always.

You finish with the adolescent, prescribe OCPs, have her come back in several
weeks to see how she is doing and to perform a pelvic examination. The teen has
private insurance and she is concerned that her parents might get a bill for her care.
Question:
Who is responsible for payment of her pelvic examination and testing?
Does the insurance company have to keep her information confidential?
Answer
The law is not always clear on responsibility of payment, but in many states this
responsibility follows the person who gives consent. Thus, if the teen's parents have
not given consent and have no information about this care, they may not be
responsible for payment.
The question of confidentiality and the insurance company is very complicated. If
the insurance is private and the parents are holders of the policy, it is probable that
in most cases, if a test is ordered, a copy of the bill may go to the parents. In cases
of care provided through public funding or HMO's this is not as likely. Thus, if the
teen wants completely confidential care in this circumstance, there are several
options:

Teen can go to a family planning or a free clinic where she might not have to
pay for services.
Teen could pay for the services herself
Teen might qualify for special public funding for reproductive health services
Teen might reconsider involvement of her parents.

Health Screening
Goal: To promote optimal physical and mental health, and to support
healthy physical, psychological, and social growth and development.
As many of the common morbidities and moralities of adolescence are related to
preventable health conditions associated with behavioral, environmental and social
causes, it is important that preventive services for this age group reflect these
issues. It is important to both reinforce positive health behaviors (e.g. exercise and
good nutrition) while discouraging potentially health-risk behaviors (e.g. unsafe
sexual practices, smoking, unsafe driving etc.). As lifetime habits may form during
this age group, it is an important time to implement health promotion and
preventive services.
Evidence based research on preventive services guidelines is only in its infancy. This
is an important area of research given the limitation on health resources and the
focus on evidence-based medicine.

COMPARISONS AMONG RECOMMENDATIONS FOR ADOLESCENT


PREVENTIVE SERVICES
(Adapted from Elster AB. Comparison of recommendations for adolescent clinical
preventive services developed by national organizations. Arch Pediatr Adolesc Med
1998;152:193. )
Subject
Immunizations
ACIP recommendations
Health guidance for teens
Normal development
Injury prevention
Nutrition
Physical activity
Dental health
Breast or testicular selfexam
Skin protection
Health guidance for
parents
Screening/counseling
Obesity
Contraception
Tobacco use
Alcohol use
Substance use
Hypertension
Depression/suicide
Eating disorders
School problems
Abuse
Hearing
Vision
Tests
Tuberculosis
Papanicolaou test
HIV infection
STDs
Cholesterol
Urinalysis
Hematocrit
Periodicity of visits
Target age group (yr)

AAFP

AAP

AMA

Yes

Yes

Yes

Yes

Yes

Yes
Yes
Yes
Yes
Yes

Yes
Yes
Yes
Yes
Yes

Yes
Yes
Yes
Yes
No

Yes
Yes
Yes
Yes
Yes

No
Yes
Yes
Yes
Yes

Yes

Yes

No

Yes

No

Yes

Yes

Yes

Yes

Yes

Yes

Yes

Yes

Yes

No

Yes
Yes
Yes
Yes
Yes
Yes
No
No
No
No
Yes
No

Yes
Yes
Yes
Yes
Yes
Yes
Yes
Yes
Yes
Yes
Yes
Yes

Yes
Yes
Yes
Yes
Yes
Yes
Yes
Yes
Yes
Yes
No
No

Yes
Yes
Yes
Yes
Yes
Yes
Yes
Yes
Yes
Yes
Yes
Yes

Yes
Yes
Yes
Yes
Yes
Yes
No
No
No
No
No
No

Yes
Yes
Yes
Yes
Yes
No
No
Tailored
13-18

Yes
Yes
Yes
Yes
Yes
Yes
Yes
Annual
11-21

Yes
Yes
Yes
Yes
Yes
No
No
Annual
11-21

Yes
Yes
Yes
Yes
Yes
No
No
Annual
11-21

Yes
Yes
Yes
Yes
No
No
No
Tailored
11-24

AAFP - American Academy of Family Physicians


AAP - American Academy of Pediatrics
AMA - American Medical Association
BF - Bright Futures
USPSTF - US Preventive Services Task Force

BF

USPSTF

Overall, the guidelines of the various groups are more similar than different. One
difference between the recommendations is the periodicity which for GAPS, BF, and
the AAP are annual visits for preventive services versus the USPSTF and AAFP which
recommend visits every 1-3 years based on the specific needs of the individual.
Blockades to preventive services to adolescents include:

The concept that adolescents are "generally healthy" and do not need
services
The reluctance of adolescents to seek care
Low reimbursement rate
Lack of confidentiality
Transportation problems
Lack of health care providers trained and interested in caring for adolescents

Solutions include a broader base of health care settings including private physicians'
offices, within health maintenance organizations (HMOs), in school-based health
clinics, in family planning clinics and in public health clinics.
Other useful information regarding prevention strategies in adolescents include:

Position Paper on Clinical preventive Services for Adolescents from Society for
Adolescent medicine: http://www.adolescenthealth.org/AM/Template.cfm?
Section=Position_Papers&Template=/CM/ContentDisplay.cfm&ContentID=146
4
The United States goal in Healthy People 2010 for adolescents at:
www.health.gov/healthypeople

Some of these include:

Consistent use of seat belts while driving


Never driving while drinking or using drugs
Consistent use of condoms if sexually active
Never smoking
Eating a prudent diet
Getting regular aerobic exercise

QUESTIONNAIRES AND OTHER HEALTH SCREENING TOOLS


There are several ways to obtain screening health information from teens

Interview during the routine examination. Use of the HEADS psychosocial


intake profile is helpful as outlined in section: A4
Screening forms - These can often complement the personal interview. The
GAPS screening forms are available at the AMA web site

Computer Aided Screening and Assessment: There has been increased


interest in this technology for assisting in screening. Further work needs to be
done on the best programs and ways to implement this technology.

The history should include

Past Medical history including


Childhood infections and illnesses
Prior hospitalizations and surgery
Significant injuries
Disabilities
Medications, including prescription medications, over-the-counter
medications, complementary or alternative medications, vitamins, and
nutritional supplements.
Allergies
Immunization history
Prior developmental history and mental health history
Family history including
Health status and age of family members
Significant physical or mental illnesses in the family
Psychosocial profile - See A4
Review of systems

The physical examination should include:

Height, Weight, Vital Signs - Body mass should be calculated.


Vision Screening
Hearing Screening
Sexual Maturity Rating
Skin exam
Teeth and Gums
Neck exam for thyromegaly or adenopathy
Cardiopulmonary
Abdominal exam
Musculoskeletal
Breast exam
Neurologic: Cursory unless specific neurologic problem
Genitalia in males and pelvic exam as indicated in females

LABORATORY TESTS
Few laboratory tests are needed to screen adolescents. These might include:

Hemoglobin: Recommended at first encounter or at the end of puberty.

Routine urinalysis recommended at first encounter with an adolescent


however up to 1/3 rd of healthy adolescents will have small amounts of
proteinuria.
Sickle cell screening in African American youth should be done if the
individual has not previously tested.
Targeted or routine cholesterol screening should be done.
Sexually active adolescents should be screened for gonorrhea and chlamydia.
If indicated by risk profile, syphilis serology and/or HIV screening should be
offered.
Tuberculosis screening with PPD should be done based on assessment of
individual risk factors and recommendations of the local health department.

IMMUNIZATIONS
An immunization history should be obtained and immunizations should be updated.
This age group still has significant rates for non-immunization. Schedules are
available from the Advisory Committee on Immunization Practices (ACIP) of the
CDC. Specific countries and areas should examine the recommendations of their
area as these vary from country to country and even state to state. The current U.S.
recommended immunization schedule is available at www.cdc.gov. In addition,
international travel information is available at: http://www.cdc.gov/travel
Potential needs in adolescents include dT booster, MMR, hepatitis A and B, and
varicella.
Part of the GAPS project has been to develop both methods of assessment and
interventions. GAPS recommends the use of the mnemonic G-A-P-S Gather information - Screen for problems
Assess further - If a problem identified then assess level and nature of risk
Problem Identification - Work with teen toward agreement on the problem and to
make changes
Specific Solutions. - This involves helping the teen with self-efficacy, giving the teen
support, solving problems in working toward a solution and shaking on a contract.
A publication from the AMA, GAPS: Clinical Evaluation and Management Handbook,
includes fully developed algorithms for each of the GAPS recommendations (
http://www.ama-assn.org/adolhlth/ama/pub/category/1947.html).

Web Sites
Available websites with preventive health guidelines include:

Department of Health and Human Services:


http://odphp.osophs.dhhs.gov
Recommendations on periodic health examinations based on the health risks of
specific age groups. The recommendations are evidence based or expert opinion
based.
GAPS - Guide to Adolescent Preventive Services - American Medical
Association
These are the comprehensive guidelines puts in place by the AMA's Division of
Adolescent Health. The recommendations are called GAPS - Guide to Adolescent
Preventive Services. GAPS recommendations cover both the content and delivery of
health care to adolescents. Several web sites are available:
For survey questionnaires: http://www.ama-assn.org/ama/pub/physicianresources/public-health/promoting-healthy-lifestyles/adolescent-health/guidelinesadolescent-preventive-services/screening-health-guidance-suicide-depression.shtml
For actual recommendations http://www.ama-assn.org/ama/pub/physicianresources/public-health/promoting-healthy-lifestyles/adolescent-health/guidelinesadolescent-preventive-services/suicide-depression-resources.shtml
Other useful links are available at the adolescent health section of AMA at
http://www.ama-assn.org/ama/pub/category/1947.html including

healthy people 2010 recommendations for adolescents

Adolescent health resources and links

Bright Futures
The Bright Futures (BF) guidelines for the health care supervision of infants,
children, and adolescents were published in 1994 and represent the work of expert
panels convened through a collaboration of the Maternal and Child Health Bureau of
the Health Resources and Services Administration, and the Medicaid Bureau of the
Health Care Financing Administration. The guidelines are both evidence-based and
based on expert opinion. They are available at:
www.brightfutures.org/
American Academy of Pediatrics
The AAP has also reviewed the preventive care for children and adolescents and
published revised recommendations in 1995. These recommendations represent a
consensus by the Committee on Practice and Ambulatory Medicine in consultation
with national committees and sections of the American Academy of Pediatrics. In
1996, the AAP also released Guidelines for Health Supervision III which more
comprehensively describes the elements of health supervision visits for children and
adolescents.
However, currently the AAP is working with Bright Futures through two cooperative
MCHB grants to help facilitate usage of Bright Futures among child health

professionals and the public. The new web site starting in June 2003 will be:
http://brightfutures.aap.org
The American Academy of Pediatrics recent guidelines are posted at
http://aappolicy.aappublications.org/practice_guidelines/index.dtl
These are from March 2000.
American Academy of Family Physicians (AAFP)
AAFP offers age-specific recommendations for periodic health examinations for
healthy patients. The AAFP recommendations are derived from the USPSTF report
by the Commission on Public Health and Scientific Affairs of the AAFP. The website is
at:
http://www.aafp.org
Also available are:
Canadian Task Force on Preventive Health Care
http://www.ctfphc.org
World Health Organization
http://www.who.int/child-adolescent-health/prevention/adolescent.htm
SEXUALITY
It is not always comfortable for a clinician to deal with sexual issues of adolescents.
Suddenly the 6 or 8 year old child that has been coming in for ear infections or
rashes is turning into an adult. In the process the teen is developing both physical
changes but is becoming much more interested and involved in their sexual identity
and relationships. An additional part of this is dealing with the consequences of
sexual behaviors including sexually transmitted diseases and pregnancy. However,
clinicians must be aware that all teenagers are sexual beings whether or not they
are sexually active and also that teens engage in sexual activities other than
vaginal intercourse. The reality is that sexual development and behavior does not
start during adolescence or adulthood, but with childhood sexual curiosity. It is
critical for health-care providers caring for adolescents to understand sexuality
during the teenage period and to be familiar with ways to deal with teenagers'
questions, feelings, and problems.
A FEW DEVELOPMENTAL ISSUES
Preadolescent period:
Biological sex is determined based on chromosomes, gonads, and hormones. In
general, gender identity or sense of masculinity and femininity is established during
this period also. During this period there is low physical and mental time spent on
sexuality issues.
Early Adolescence

This period is characterized by:


Early pubertal developmental changes
Curiosity and concern over one's body and one's peers
Sexual fantasies are common as well as beginning of masturbation activity
Most sexual activity is nonphysical such as phone conversations

Middle Adolescence
This period is characterized by:

Full physical maturation including menstruation in females


High sexual energy with more emphasis on physical contact
Sexual exploration activity including dating, kissing, casual relationships of
both coital and noncoital nature
Denial of consequences of sexual behaviors
Late Adolescence
This period is characterized by:

More expressive and less exploitative sexual behaviors


More intimate sharing relationships

Adolescents are filled with questions about their sexuality including?

Am I normal?
Is masturbation ok?
Am I ready for a sexual relationship or intercourse?
How do I say no?
What is safe sex?
What is contraception?
Am I gay?

SEXUAL BEHAVIORS
Given the need, do physicians address issues of adolescent sexuality?
In a recent CDC news release (PACT5, December 8, 2000) it was found that in a
survey of 15,000 high school students from the U.S., only 43% of teenage females
and 26% of teenage males discuss pregnancy or sexually transmitted infections
with their physicians during routine exams.
UNWANTED SEXUAL EXPERIENCES
Unfortunately not all adolescent sexual involvement is consensual.

Over 80% of females in grades 8-11 and over 2/3 of males experienced
unwanted sexual comments or actions in 1993.
Sexual intercourse in young adolescents in particular may not be voluntary.
Data presented by the Alan Guttmacher Institute indicates that about 74% of
women who had intercourse before age 14 and 60% of those who had sex
before age 15 report having had sex involuntarily (Alan Guttmacher Institute,
1994

Suggestions to help adolescents better deal with their sexuality include:

Listening to teen's feeling and concerns and tempering ones own reactions.
Parents can exert a strong positive influence, not through moralizing,
lecturing, or invasion of privacy, but through helping the adolescent in his or
her decision-making process.
Timing: Because sexuality begins in childhood, it is important to treat
sexuality as a natural part of life from birth onward. Given this perspective, it
is much less awkward to have discussions about sexuality when children grow
up.
Education: Adolescents should be informed and knowledgeable -with the aid
of parents, school, or community resources in areas including basic
reproductive anatomy and physiology, basic sexual functioning, health
consequences of sexual intercourse, decision making skills
Avoid joking about sexuality
Admit personal discomfort
Have available resources including books and pamphlets or web sites.
Respect the adolescent's privacy
Be aware of community resources.
Web Sites Sites for Teenagers and parents

www.goaskalice.columbia.edu
Go Ask Alice ! . Source of general health and sex information maintained by
Columbia University health educators. Most questions answered are submitted by
high school and college-aged people.
www.iwannaknow.org
This web page is specifically designed for teenagers to find answers to their
questions about their bodies, sex, and sexual feelings, and to provide them with
responsible educational information in a relaxed, safe, and fun environment.
http://www.teenwire.com/index.asp
TeenWire. Sponsored by the Planned Parenthood Federation of America . Provides
teens with unbiased, uncensored sexuality and sexual health information.
www.unicef.org/voy
Voices of Youth. UNICEF. Designed for youth worldwide as a venue to share ideas.

www.ippf.org
International Planned Parenthood Federation (IPPF) . Links family planning
associations in over 150 countries worldwide. Provides information to a number of
other sites.
www.paho.org
Pan American Health Organization (PAHO). . Address the health of adolescents and
youth within the context of their social and economic environment.
http://www.plannedparenthood.org
Planned Parenthood Federation of America .
www.popcouncil.org
The Population Council. . Organization conducts reproductive health research and
policy work worldwide. Publications cover a range of reproductive health topics,
including adolescent health.
http://www.siecus.org
Sexuality Information and Education Council of the U.S. , SIECUS Home Page . 2000,
SIECUS. SIECUS develops, collects, and disseminates information, promotes
comprehensive education about sexuality, and advocates the right of individuals to
make responsible sexual choices.
MEDICAL PROBLEMS
ABDOMINAL PAIN
Chronic abdominal complaints are a frequent concern or complaint of adolescents
and young adults. One definition is three or more separate episodes of pain that
occur over a 3 - month period. In most cases of recurrent abdominal pain in
adolescents, no specific organic problem is found. The prevalence is as high as 510% or more of all adolescents.
Differential Diagnosis includes:

Functional abdominal pain often related to stress and eating habits. The pain
tends to be periumbilical, crampy and nonspecific without radiation. It usually
does not wake adolescents. There may be associated nausea and vomiting,
headaches, fatigue, dizziness and diarrhea. It does not usually cause weight
loss or other systemic symptoms. It distinction organic abdominal pain
usually includes more localized pain and may awake the teen from sleep.
Irritable Bowel Syndrome : Pain is usually colicky in nature and is usually
more common in older adolescents and more common in females.
Lactose intolerance which is associated with crampy abdominal pain,
diarrhea, flatulence and belching
Gynecologic conditions such as ectopic pregnancy, mittleschmerz, ruptured
ovarian cysts and pelvic inflammatory disease.
Musculoskeletal conditions like costochondritis or muscle wall strain
Hepatitis and pancreatitis

Gastrointestinal infections such as giardiasis


Referred pain from lower lungs such as pneumonia or spinal cord tumor
Gastrointestinal disease such as peptic ulcer disease, inflammatory bowel
disease or obstructed bowel.
Systemic conditions : Occasionally systemic conditions in adolescents may
lead to abdominal pain such as diabetic ketoacidosis, sickle cell crisis,

Diagnosis
An organic disease is usually suggested by the history, physical examination and
results of screening laboratory tests. The history should include pain description,
family history, current stresses and relationship to pain. It may be helpful to have
teen keep a pain and dietary diary. The examination should include height and
weight and growth charts, careful examination of abdomen for tenderness, rebound,
hepatosplenomegaly or masses. Signs of systemic diseases should be looked for
and a pelvic examination if indicated. Screening laboratory tests include CBC,
sedimentation rate, urinalysis, basic chemistry panel and liver enzymes. In addition,
stool samples for occult blood, ova and parasite may be needed. Other helpful tests
might include stool alpha-antitrypsin test as screen for IBD or protein losing
enteropathy as well as plain film of abdomen and H.pylori antibody titer. More
complicated or invasive tests might be needed depending on initial evaluation.
If the diagnosis of functional abdominal pain is made, the clinician will need to
explain to the need the meaning of this disorder. In addition, the clinician needs to
explain that the symptoms are real but can result from emotions and feelings. The
clinician can use the example of blushing, a physiological response to the feeling of
embarrassment. The teen should be reassured that they can return to their
activities and school.
CHEST PAIN
As many as 5% of adolescents in medical clinics complain about chest pain. In
contrast to adults, acute chest pain in adolescents is rarely of cardiac origin.
However, many teens fear having a heart attach or having cancer. The common
causes of chest pain in adolescents includes:

Musculoskeletal including precordial "stitch", muscle strain, costochrondritis,


Tietze's syndrome and much less commonly slipping rib syndrome,
fibromyalgia, thoracic outlet obstruction and metatstatic bone disease.
Psychogenic including stress, hyperventilation and depression
Pulmonary causes including cough, asthma, pneumonia, pleural effusion,
pleurodynia, pneumothorax, acute chest syndrome with sickle cell disease
and acute pulmonary embolism.
Gastrointestinal including reflux, peptic ulcer disease, gastritis, cholecystitis
Trauma to ribs
Breast lesions or mastitis
Cardiac conditions such as mitral valve prolapse, pericarditis, myocarditis and
rare congenital problems

Less common problems also include herpes zoster

The diagnosis is usually based on history and physical examination. Important


historical items include characterization of pain, precipitating and alleviating
factors, recent activity, trauma, recent infections, associated symptoms and recent
stress. Physical examination includes vital signs, chest wall palpation,
cardiopulmonary examination, breast examination and abdominal examination.
Most adolescents will not require any further laboratory tests and usually an
electrocardiogram and chest radiograph are normal. Symptoms that should be of
particular concern to the clinician include acute chest pain precipitated by exercise,
pain that interferes with sleep or associated with dyspnea, palpitations, dizziness or
syncope.
FATIGUE
Another common complaint among teens is fatigue. This may be a common
complaint from parents who may be concerned that their teens seem to not be
doing enough. It is less common to have teens complain of severe fatigue. The most
common cause of fatigue in teens is nonorganic representing a reaction to stress,
anxiety or depression. Causes include psychosocial causes (stress, anxiety,
depression), inadequate sleep, dieting, pregnancy, medications, infections,
allergies, systemic diseases (renal, anemia, malignancy, collagen vascular diseases,
thyroid dysfunction, diabetes mellitus, IBD) and chronic fatigue syndrome.
The evaluation should include careful review of systems, medical history,
psychosocial history including alcohol and substance use and sleep history. History
suggestive of organic causes include fatigue that increases during the day, that
decreases with rest, history of fever, weight loss, night sweats, arthritis or
lymphadenopathy. The physical examination may point to an organic problem. Many
teens will not require any laboratory tests but if there is any questions, a screening
evaluation might include, CBC with differential, urinalysis, mononucleosis test,
sedimentation rate and perhaps a screening chemistry panel. Other tests would be
based on the history and examination.
Chronic fatigue syndrome
Chronic fatigue syndrome (CFS) is a clinically defined syndrome for adults that is
characterized by new onset, severe, disabling fatigue and a combination of
symptoms highlighted by self-reported impairments in concentration and short-term
memory, musculoskeletal or joint pains, sleep disturbances, headaches, sore throat,
tender lymph nodes and post exertional malaise. Diagnosis excludes uncontrolled
chronic illness, past or current mental illnesses like depression, bipolar affective
disorder or anorexia nervosa. The criteria for adults can be found at the CDC web
site at http://www.cdc.gov/ncidod/diseases/cfs and then going to the CFS definition.
However, there are no accepted criteria for CFS in adolescents. The etiology and
pathophysiology is controversial and unknown. It has been linked to various viruses
and may be associated with various immunological abnormalities. The evaluation is
similar to that discussed for general fatigue. Treatment has involved reassurance,
low-dose antidepressants, psychotherapy and physical therapy.

HEADACHES
Recurrent headaches are also a frequent problem in adolescents and young adults.
Almost 75% of teens by age 15 have experienced headaches. Most recurrent
headaches in adolescents and young adults are not associated with severe organic
pathology. However, they may be signs of stress, anxiety, or depression. This is in
contrast to a isolated single very severe acute headache that may be a sign of
organic disease. Most headaches are a result of either vascular dilation, muscular
contraction, traction of structures or local inflammation.
Epidemiology
By age 12 about 66% of adolescents have had headaches and this increases to 75%
by age 15. About 25% of migraine headaches begin during childhood and
adolescence. After age 12 headaches become more common in females.
Differential Diagnosis

With acute severe headache


Febrile patients: meningitis, brain abscess, sinusitis, other infections
Afebrile patient: Subarachnoid hemorrhage, intracerebral hemorrhage, postseizure headache, severe hypertension, acute dental disease, or acute orbital
disease
With episodic, recurrent headaches and complete recovery between episodes
Muscle tension type headaches - Associated with bandlike, bilateral, steady
pain and usually lack nausea, vomiting, photophobia or neurologic symptoms
Classic Migraine: Classic migraine is associated with aura, unilateral
throbbing headache and also nausea and/or vomiting. Photophobia, family
history and history of motion sickness are common.
Common migraine is similar to classic but lacks aura and may be bilateral.
Migraine variants include hemiplegic migraine, confusional migraine,
abdominal migraine and ophthalmoplegic migraine.
Cluster headaches - Associated with burning or pain behind one eye with
sudden onset also rhinorrhea, lacrimation and conjunctival injection on same
side.
Chronic headaches but continuous or increasing in intensity after onset
Intracranial mass lesions, hydorcephalus, post-lumbar puncture headaches,
pseudotumor cerebri, depressive headaches, post-trauma, local extracranial
disease, pregnancy, chronic meningitis, substance abuse, obstructive sleep
apnea.

Diagnosis
In diagnosing the cause of headaches, the history is the primary diagnostic tool with
examination being also key. The history should include onset, pattern and
chronology of the pain, associated symptoms, preceding symptoms or visual
symptoms, precipitants including stress, illnesses, foods, medication and caffeine.
Medications can be important including analgesics, birth control pills and

tetracycline. Substance abuse history and stress history is important as well as


history of migraines in the family. The physical examination includes a good general
examination with a careful neurological examination. In general, teens with
recurrent headaches and separated by periods of complete recovery rarely need
further laboratory evaluation. Neuroimaging is indicated in the acute severe
headache or increasing constant headache or teens with abnormal neurological
examination.
Therapy
It is generally better to take medications sooner in the onset of the headache than
later. Reassurance in most teens and families is a key issue. A headache diary can
be helpful in eliminating triggering events or foods. Helpful interventions in tension
headaches include relaxation exercises, simple analgesics or combined analgesics
with both acetaminophen and nonsteroidal anti-inflammatory medications. Migraine
headaches may respond to elimination of certain triggering items as well as
stabilizing caffeine intake. Medications include simple analgesics, antiemetics,
sedative-analgesic combinations, ergot derivatives, and the triptan medications for
acute severe migraine headaches. Prophylactic treatment can include beta-blockers,
antidepressant medications, low dose non-steroidal medications, anticonvulsants
(valproic acid and phenytoin), calcium channel blockers and clonidine.
SLEEP DISORDERS
Sleep problems can be a common problem in teens as either a major complaint or
on the review of systems. Problems can include insomnia, hypersomnia (narcolepsy
and excessive daytime sleepiness) and parasomnias (nightmares, night terrors,
sleepwalking and nocturnal enuresis). Adolescents require about 8 1/2 to 9 1/2
hours of sleep per night but actually get far less. Early teens sleep about 9 hours,
mid adolescents about 7 1/2 hours and late adolescents about 7 hours. Teens with
sleep problems should be asked about the type of problem, frequency, duration,
daytime symptoms, family history, age of onset, bedtime habits, prior treatment,
psychosocial history and medications and drug history.
The most frequent problem in teens is insomnia involving either trouble falling
asleep, staying asleep or awakening too early. Treatment involves identifying any
organic problems and psychosocial stresses. Important interventions include
counseling, regularizing bedtime hours, relaxation techniques, daily exercise,
curtailing caffeine and alcohol and avoid daytime naps.
Questions
Question #1
What is the most common cause of recurrent abdominal pain in adolescents?
Answer #1
Functional abdominal pain

Question #2
Name 5 other relatively common causes of recurrent abdominal pain in
adolescents?
Answer #2

Irritable bowel syndrome


Lactose intolerance
Muscle wall inflammation or trauma
Hepatitis
Gynecologic infections
Rupture ovarian cysts
Inflammatory bowel disease

Question #3
What are the most common causes of chest pain in adolescents?
Answer #3

Chest wall musculoskeletal strain or trauma


Stress
Hyperventiliation
Cough

Question #4
A teen presents with 3 months of fatigue. What would be important findings on
history and examination that would suggest this is organic?
Answer #4

History of systemic symptoms


Fatigue increases throughout the day
Fever
Weight loss
Focal examination findings such as arthritis, lymphadenopathy,
hepatospenomegaly

Question #5
What are important history and physical findings that would suggest a serious cause
of headaches?
Answer #5

Acute onset of severe headache

Headache that is constant, persistent is increasing in intensity


Focal neurologic symptoms such as weakness, motor changes or seizures
Focal neurologic signs on examination
Abnormal fundoscopic examination
Absence of findings suggestive of migraines such as aura, photophobia
Persistent vomiting
Cases

Case #1
A 16 year old female complains of about 5 months of occasional abdominal pain.
The pain is crampy in nature. What would be key history questions in this teen?
Answer:

Nature of pain including severity, radiation and precipitating or alleviating


factors
The pain occurs in lower abdomen about once or twice a month. There is no
radiation. It is not increased or decreased with food, bowel movements or
exercise. The pain does not wake her up at night.
Associated symptoms including nausea, vomiting, diarrhea or constipation
There is not history of any of these symptoms
Menstrual history and sexual history
Confidentiality should be discussed before taking the sexual history. She has
a boyfriend and they have been having sexual intercourse for six months with
occasional use of condoms. Her menses started at age 12 and has regular for
several years. Her last menstrual period was about 4-5 weeks ago and her
last sexual intercourse was two weeks ago. She denies any vaginal discharge
or genital lesions. The pain appears to probably come most months in mid
cycle.
Review of systems
She denies any other systemic complaints and review of systems is negative.
Psychosocial history
She lives with her parents and they get along well. She denies any use of
drugs and she denies and mental health issues.

Question
What would be important parts of her physical examination?

Vitals signs:
BP 110/76, Respirations 12, Temperature 37, Her weight and height are 45 th
percentile
Abnormalities on skin, joint, cardiopulmonary exam to suggest chronic
disease:
These are all normal
Abdominal examination
There is no tenderness, no organomegaly and bowel sounds are normal

Pelvic examination
Because the adolescent is sexually active, she is not contracepting well, and
because she has abdominal complaints, a pelvic examination should be
performed. It could either be performed at that time or in the very near
future. Because she has no acute symptoms and because she has no vaginal
complaints and she has no abdominal tenderness, if it could not be
performed at that time, it could be rescheduled for near future. This would
also depend on the ease of having the teen return.

The pelvic examination shows no external genital lesions, no discharge, no cervical


motion tenderness, uterine or adnexal tenderness or masses.
Question
What would you do at this point in time?
Answer
There are several issues at the present time. First, she has the history of abdominal
pain, second she is sexually active without contraception and is at risk for both STIs
and pregnancy, third is the issue of a possibility of delayed menses and pregnancy.
In regards to her abdominal pain, one possibility would be mittleschmerz. The pain
seems to come mainly at the time of mid-cycle, although the history is not entirely
clear. A menstrual calendar with a diary of her pain might help diagnosis this or
establish if there are other correlations to her pain. There would appear to be no
history to suggest serious organic disease or systemic disease.
In regards to her reproductive issues, it would be important to discuss with her the
potential issues of pregnancy and STIs and explore what options she would like to
go in. It would be important to explore the nature of her relationship including age
of partner, consensuality of the intercourse. It would be important to test for
chlamydia and gonorrhea during the pelvic examination. A pap smear could be done
at this point although, recommendations are moving into possibly waiting until three
years after the onset of sexual activity to perform first pelvic exam.
This adolescent has only occasionally used contraception and may be late with her
menses. It would also be important to discuss the possibility of pregnancy and to
obtain a pregnancy test. It would also be important to discuss more reliable forms of
contraception and other options including hormonal contraceptive options and more
consistent use of condoms.
Case #2
A 15 year old male complains of about 6 months of recurrent abdominal pain. The
pain is crampy in nature. On history you find that:
Nature of pain: The pain occurs in lower abdomen and midabdomen several times a
month. It may last from hours to on and off for several days. There is no radiation. It
sometimes is worse with food intake but is not associated with exercise. Bowel
movements sometimes help. The pain does not wake her up at night.

Associated symptoms : There is no history of nausea, vomiting. There is occasional


history of constipation.

Sexual history: Confidentiality is discussed. The teen is not sexually active


and has no sexual relationships.
Review of systems : Occasionally has headaches and occasional trouble
getting to sleep.
Psychosocial history : The teen lives with his mother has mom is divorced. He
states he fights with her all the time as she does not let him "live his own
life". He feels she is watching over him all the time. He has been doing poorly
in school with a below average grades that have worsen in past year. He
admits to drinking heavily on the weekends and occasionally using other
drugs. He has tried marijuana. He occasionally get very depressed and has
once or twice thought of taking a bottle of the pills his mother keeps in her
cabinet.

Examination :

Vitals signs: BP 120/80 Respiration 16, Temperature 37, His weight and height
are 55 th percentile
There are no abnormalities on skin, joint, cardiopulmonary exam to suggest a
chronic disease:
Abdominal examination: There is minimal tenderness in midabdomen and no
organomegaly and bowel sounds are normal. There are no masses. There is
no rebound tenderness.

Question
What would be your next steps?
Answer
There are several issues significant issues at this point. First is the abdominal pain
that the teen is concerned about. In addition, there appears to be some significant
conflict between the teen and his mother. The teen is also doing poorly in school
and this has worsen. In addition, he appears to be drinking heavily and has a history
of depression and perhaps suicidal ideation.
Regarding the abdominal pain. There is nothing on the history to suggest an organic
etiology and there is much to suggest that the pain may be functional and stress
related. However, some basic screening laboratory might be in order including a
CBC, sedimentation rate and perhaps a screening chemistry panel.
Blood tests show normal CBC, sed rate of 10 mm/hour and no abnormalities on
chemistry panel.
Regarding his psychosocial history: It would be important to get a complete history
from the mother on her perspectives on both the abdominal pain and her thoughts
on how things are going with her son. It would also be important to evaluate how
the two react together, so bringing the mom in and reviewing the evaluation of the
abdominal pain with the two together may be helpful. Obviously there are

significant issues with the teen and the family unit. It would be important to assess
with the teen alone his degree of suicidal ideation. If very high, then an immediate
referral would be necessary. In this case, there is no current desire to hurt himself
and no current or past plan or attempts. You discuss that with the teen the
probability that the pain is related to some of the difficulties he is experiencing and
that some additional help with his relationship with his mother might be helpful to
sort things out. While initially somewhat reluctant he is willing to see your colleague
for individual and family sessions. These are scheduled as well as scheduling a
follow-up with the teen for his abdominal pain. He is also asked to keep a diary of
his pain.
Case #3
A 15 year old comes in complaining of 3 months of occasional chest pain.
Question
What would be the important historical questions in this teen?

Answer
Characterization of pain
The pain is a sharp but occasionally aching pain in anterior chest. The pain
does not prevent him from doing his normal activities. The pain is unrelated
to exercise or meals but his occasionally worse on movement or turning his
body.
Precipitating and alleviating factors
Unrelated to food intake. It is worse with coughing or deep breathing. It does
not awaken the teen at night.
Recent trauma
There is no history of any recent trauma
Recent infections or systemic illness
There is no history of any recent infections, the teen has no serious illnesses,
no asthma and no history of cough.
Medications or drugs
Teen is on no medications and has no used any illicit drugs. Teen does not
smoke.
Associated symptoms
There is no history of shortness of breath, dyspnea on exertion, wheezing,
syncope, lightheadedness or paresthesias.
Family history
There is no family history of cardiovascular diseases.
Recent stress
There is no history of recent stress and teen is doing well at home and at
school.

Question
What are important things to be checking on physical examination:

Answer

General state
Teen is in no acute distress and appears healthy
Vital signs
BP 110/80, respirations 12, pulse: 80, height and weight: 40 th percentile
Chest wall palpation
There is slight tenderness along the left costochondral junction at about the
third and fourth ribs. There is no swelling or masses. There is no evidence of
trauma.
Cardiopulmonary examination
The pulmonary exam is normal with normal bilateral breath sounds and no
rales or rubs. The heart sounds are normal with no murmurs or clicks.
Breast exam
There is no gynecomastia and no breast tenderness
Abdominal examination
There is no tenderness or masses or organomegaly

Question
What is the most likely diagnosis?
Answer
Given the negative history to suggest any chronic disease, the negative history of
trauma or infections, the negative history of association with exercise but some
increase with movement and the exam showing tenderness at the costochondral
junction, the most likely diagnosis is costochondritis.

Вам также может понравиться